Matemática, perguntado por fernandasilva36, 1 ano atrás

O conjunto solução, em R, da inequação (1 – log 2) . log5 x ≤ log 3 – log (x – 2) é
a.
]– ∞; 3]
b.
]2; 3]
c.
]2; 5]
d.
[4; 6]
e.
[3; + ∞[

Soluções para a tarefa

Respondido por luanarbeletcheoym1rl
5
Olá!

Se eu entendi corretamente, a inequação é a seguinte:

(1-\text{log}2)\text{log}_5x\leq\text{log}3-\text{log}(x-2)

Para resolver essa inequação, vamos utilizar algumas propriedades dos logaritmos. Primeiro, vamos escrever:

1=\text{log}10

Com isso, temos:

(\text{log}10-\text{log}2)\text{log}_5x\leq\text{log}3-\text{log}(x-2)

Agora, vamos utilizar o fato de que a subtração de logaritmos é igual ao logaritmo da divisão. Isto é, temos que:

\text{log}a-\text{log}b=\text{log}\frac{a}{b}

Assim, podemos escrever:

(\text{log}10-\text{log}2)\text{log}_5x\leq\text{log}3-\text{log}(x-2)
\text{log}\frac{10}{2}\cdot\text{log}_5x\leq\text{log}\frac{3}{x-2}
\text{log}5\cdot\text{log}_5x\leq\text{log}\frac{3}{x-2}

Existe outra propriedade dos logaritmos que nos permite fazer uma mudança de base. A saber:

\text{log}_ba=\frac{\text{log}_ca}{\text{log}_cb}

Com isso, obtemos:

\text{log}5\cdot\text{log}_5x\leq\text{log}\frac{3}{x-2}
\text{log}5\cdot\frac{\text{log}x}{\text{log}5}\leq\text{log}\frac{3}{x-2}
\text{log}x\leq\text{log}\frac{3}{x-2}

Agora, repare na inequação acima. Quando x<2, o argumento do logaritmo à direita se torna negativo. Como sabemos, a função logaritmo não admite argumentos negativos (ou zero). Daqui, depreendemos, portanto que x>2!

Agora, com base na inequação anterior, podemos escrever:

x\leq\frac{3}{x-2}
x(x-2)\leq{3}
x^2-2x-3\leq{0}

Vamos determinar as raízes da função de segundo grau utilizando a fórmula de Báskara:

x=\frac{2\pm\sqrt{(-2)^2-4\cdot{1}\cdot(-3)}}{2}
x=\frac{2\pm\sqrt{4+12}}{2}
x=\frac{2\pm\sqrt{16}}{2}
x=\frac{2\pm{4}}{2}
x=1\pm{2}

Logo, as raízes são:

x'=3
x''=-1

Olhe para o gráfico da função que eu anexei. Temos que à esquerda de -1, a função é positiva e, nesse caso, nossa inequação não se cumpre. Entre -1 e 3, a função é negativa, estando de acordo com a nossa inequação. Acima de 3, a função volta a ser positiva e a inequação não é válida.

Portanto, temos que o domínio de x deve estar entre -1 e 3. Porém lembremos do que discutimos anteriormente: x deve ser maior do que 2, caso contrário estaríamos calculando o logaritmo de um número negativo, o que não existe. Assim, podemos dizer que o conjunto solução da inequação dada é:

]2; 3]
Anexos:

fernandasilva36: Muito obrigada!
Perguntas interessantes